Laurent expansion of the given function












0












$begingroup$


I have the following question :
Find a Laurent expansion of $f(z) = dfrac{z}{(z^2 + 1)}$, valid for $|z-3| > 2$.



I have learnt finding the Laurent expansion of functions where the denominator is of the form $(z+a)^n$, but this form in the given question seems to be unfamiliar to me. I guess I need to transform it to the $(z+a)^n$ form, but how do I utilise the condition given (that it is valid for $|z-3| > 2$) ?



Thanks in advance.










share|cite|improve this question











$endgroup$












  • $begingroup$
    That question makes no sense. The poles of $f$ are $pm i$ and the distance from $3$ to $pm i$ is $sqrt{10}>2$. So, there is no single Laurent series that works in the region $lvert z-3rvert>2$.
    $endgroup$
    – José Carlos Santos
    Dec 5 '18 at 6:42












  • $begingroup$
    May I then assume that the condition given in the question has been misprinted?
    $endgroup$
    – Jasmine
    Dec 5 '18 at 7:11










  • $begingroup$
    It sure looks like.
    $endgroup$
    – José Carlos Santos
    Dec 5 '18 at 7:25
















0












$begingroup$


I have the following question :
Find a Laurent expansion of $f(z) = dfrac{z}{(z^2 + 1)}$, valid for $|z-3| > 2$.



I have learnt finding the Laurent expansion of functions where the denominator is of the form $(z+a)^n$, but this form in the given question seems to be unfamiliar to me. I guess I need to transform it to the $(z+a)^n$ form, but how do I utilise the condition given (that it is valid for $|z-3| > 2$) ?



Thanks in advance.










share|cite|improve this question











$endgroup$












  • $begingroup$
    That question makes no sense. The poles of $f$ are $pm i$ and the distance from $3$ to $pm i$ is $sqrt{10}>2$. So, there is no single Laurent series that works in the region $lvert z-3rvert>2$.
    $endgroup$
    – José Carlos Santos
    Dec 5 '18 at 6:42












  • $begingroup$
    May I then assume that the condition given in the question has been misprinted?
    $endgroup$
    – Jasmine
    Dec 5 '18 at 7:11










  • $begingroup$
    It sure looks like.
    $endgroup$
    – José Carlos Santos
    Dec 5 '18 at 7:25














0












0








0





$begingroup$


I have the following question :
Find a Laurent expansion of $f(z) = dfrac{z}{(z^2 + 1)}$, valid for $|z-3| > 2$.



I have learnt finding the Laurent expansion of functions where the denominator is of the form $(z+a)^n$, but this form in the given question seems to be unfamiliar to me. I guess I need to transform it to the $(z+a)^n$ form, but how do I utilise the condition given (that it is valid for $|z-3| > 2$) ?



Thanks in advance.










share|cite|improve this question











$endgroup$




I have the following question :
Find a Laurent expansion of $f(z) = dfrac{z}{(z^2 + 1)}$, valid for $|z-3| > 2$.



I have learnt finding the Laurent expansion of functions where the denominator is of the form $(z+a)^n$, but this form in the given question seems to be unfamiliar to me. I guess I need to transform it to the $(z+a)^n$ form, but how do I utilise the condition given (that it is valid for $|z-3| > 2$) ?



Thanks in advance.







laurent-series






share|cite|improve this question















share|cite|improve this question













share|cite|improve this question




share|cite|improve this question








edited Dec 5 '18 at 6:40









Yadati Kiran

1,751619




1,751619










asked Dec 5 '18 at 6:31









JasmineJasmine

283




283












  • $begingroup$
    That question makes no sense. The poles of $f$ are $pm i$ and the distance from $3$ to $pm i$ is $sqrt{10}>2$. So, there is no single Laurent series that works in the region $lvert z-3rvert>2$.
    $endgroup$
    – José Carlos Santos
    Dec 5 '18 at 6:42












  • $begingroup$
    May I then assume that the condition given in the question has been misprinted?
    $endgroup$
    – Jasmine
    Dec 5 '18 at 7:11










  • $begingroup$
    It sure looks like.
    $endgroup$
    – José Carlos Santos
    Dec 5 '18 at 7:25


















  • $begingroup$
    That question makes no sense. The poles of $f$ are $pm i$ and the distance from $3$ to $pm i$ is $sqrt{10}>2$. So, there is no single Laurent series that works in the region $lvert z-3rvert>2$.
    $endgroup$
    – José Carlos Santos
    Dec 5 '18 at 6:42












  • $begingroup$
    May I then assume that the condition given in the question has been misprinted?
    $endgroup$
    – Jasmine
    Dec 5 '18 at 7:11










  • $begingroup$
    It sure looks like.
    $endgroup$
    – José Carlos Santos
    Dec 5 '18 at 7:25
















$begingroup$
That question makes no sense. The poles of $f$ are $pm i$ and the distance from $3$ to $pm i$ is $sqrt{10}>2$. So, there is no single Laurent series that works in the region $lvert z-3rvert>2$.
$endgroup$
– José Carlos Santos
Dec 5 '18 at 6:42






$begingroup$
That question makes no sense. The poles of $f$ are $pm i$ and the distance from $3$ to $pm i$ is $sqrt{10}>2$. So, there is no single Laurent series that works in the region $lvert z-3rvert>2$.
$endgroup$
– José Carlos Santos
Dec 5 '18 at 6:42














$begingroup$
May I then assume that the condition given in the question has been misprinted?
$endgroup$
– Jasmine
Dec 5 '18 at 7:11




$begingroup$
May I then assume that the condition given in the question has been misprinted?
$endgroup$
– Jasmine
Dec 5 '18 at 7:11












$begingroup$
It sure looks like.
$endgroup$
– José Carlos Santos
Dec 5 '18 at 7:25




$begingroup$
It sure looks like.
$endgroup$
– José Carlos Santos
Dec 5 '18 at 7:25










0






active

oldest

votes











Your Answer





StackExchange.ifUsing("editor", function () {
return StackExchange.using("mathjaxEditing", function () {
StackExchange.MarkdownEditor.creationCallbacks.add(function (editor, postfix) {
StackExchange.mathjaxEditing.prepareWmdForMathJax(editor, postfix, [["$", "$"], ["\\(","\\)"]]);
});
});
}, "mathjax-editing");

StackExchange.ready(function() {
var channelOptions = {
tags: "".split(" "),
id: "69"
};
initTagRenderer("".split(" "), "".split(" "), channelOptions);

StackExchange.using("externalEditor", function() {
// Have to fire editor after snippets, if snippets enabled
if (StackExchange.settings.snippets.snippetsEnabled) {
StackExchange.using("snippets", function() {
createEditor();
});
}
else {
createEditor();
}
});

function createEditor() {
StackExchange.prepareEditor({
heartbeatType: 'answer',
autoActivateHeartbeat: false,
convertImagesToLinks: true,
noModals: true,
showLowRepImageUploadWarning: true,
reputationToPostImages: 10,
bindNavPrevention: true,
postfix: "",
imageUploader: {
brandingHtml: "Powered by u003ca class="icon-imgur-white" href="https://imgur.com/"u003eu003c/au003e",
contentPolicyHtml: "User contributions licensed under u003ca href="https://creativecommons.org/licenses/by-sa/3.0/"u003ecc by-sa 3.0 with attribution requiredu003c/au003e u003ca href="https://stackoverflow.com/legal/content-policy"u003e(content policy)u003c/au003e",
allowUrls: true
},
noCode: true, onDemand: true,
discardSelector: ".discard-answer"
,immediatelyShowMarkdownHelp:true
});


}
});














draft saved

draft discarded


















StackExchange.ready(
function () {
StackExchange.openid.initPostLogin('.new-post-login', 'https%3a%2f%2fmath.stackexchange.com%2fquestions%2f3026723%2flaurent-expansion-of-the-given-function%23new-answer', 'question_page');
}
);

Post as a guest















Required, but never shown

























0






active

oldest

votes








0






active

oldest

votes









active

oldest

votes






active

oldest

votes
















draft saved

draft discarded




















































Thanks for contributing an answer to Mathematics Stack Exchange!


  • Please be sure to answer the question. Provide details and share your research!

But avoid



  • Asking for help, clarification, or responding to other answers.

  • Making statements based on opinion; back them up with references or personal experience.


Use MathJax to format equations. MathJax reference.


To learn more, see our tips on writing great answers.




draft saved


draft discarded














StackExchange.ready(
function () {
StackExchange.openid.initPostLogin('.new-post-login', 'https%3a%2f%2fmath.stackexchange.com%2fquestions%2f3026723%2flaurent-expansion-of-the-given-function%23new-answer', 'question_page');
}
);

Post as a guest















Required, but never shown





















































Required, but never shown














Required, but never shown












Required, but never shown







Required, but never shown

































Required, but never shown














Required, but never shown












Required, but never shown







Required, but never shown







Popular posts from this blog

To store a contact into the json file from server.js file using a class in NodeJS

Redirect URL with Chrome Remote Debugging Android Devices

Dieringhausen